PT2.S2.Q23 - NA Question

cat_kimballcat_kimball Core Member

Hi guys,

I've been using the negation test as I go through the answer choices. I've been able to pinpoint why E is correct and why most of the other answer choices are wrong, except for answer choice C. I don't understand what I'm missing, but if someone could explain why negating answer choice C does not wreck the argument, that would be great.

Conclusion - Cause of increased deaths is bronchial inhalers by asthma sufferers to relieve their symptoms.
Premises - Two explanations for increasing death rate of people with asthma: the reporting is more accurate now and there is an increase in urban pollution.
Gap - Bronchial inhalers is not mentioned in the two premises as an explanation for increased death rate.
Answer choice C - "Bronchial inhalers are unsafe, even when used according to the recommended instructions." Why is the incorrect? If the inhalers are not unsafe, then wouldn't this wreck the argument that the inhalers cause the increased death rate? Isn't it also a must be true that the inhalers have to be unsafe for the death rate of the asthma population to significantly increase?

Comments

  • lsat_suslsat_sus Core Member
    edited August 2021 1417 karma

    My 2 cents on everything you wrote.

    1. "Unsafe" is too ambiguous of a term to be relevant or necessary imo.. It's "unsafe" to inhale carbon monoxide, but it's also unsafe to eat without washing your hands. When the stimulus says "it's unsafe even in recommended doses" - how unsafe is it? Unsafe enough to cause asthma? or maybe just a tad unsafe to cause a cough? The point is that the meaning of "unsafe" has different degrees and "unsafe enough to cause asthma" is only one of a plethora of interpretations, making it unnecessary.

    2. When you negated it - "inhalers are not unsafe/safe even in recommended doses" - what do they mean "safe?" To what degree? How safe? In order to wreck it, we need to assume that it's safe enough that it didn't cause the asthma. But it could also just be "safe" in the sense that it's "safer" compared to... idk some other product that causes an asthma at 10x the magnitude. Bad example, I know, but the idea, again, is that it's ambiguous. The fact that I can even come up with an outrageous abstract counter shows it's unnecessary. It's much harder/maybe impossible, logically, to counter AC E for example.

    3. Comment on your "Gap" and AC E - I agree that it's not mentioned, but you may have to take it one step further. Not only is it not mentioned in the two premises, but it also assumes and limits the world so that there are only three potential causes.

    In other words, he's saying
    "we have more people dying from asthma"
    It's not A (medical recording error).
    It's not B (pollution).
    Therefore it's C (bronchial inhalers).

    They assume that you can only explain the increased asthma with A, B and C.

    You'll see this pattern occur very, very frequently, especially on the parallel flaw questions (from my view though not limited to parallel flaw). You asked a fantastic question and it has supplemented my study as well! Please do continue posting these questions whenever you get a chance cuz I'll be looking out for them!

  • cat_kimballcat_kimball Core Member
    86 karma

    Thank you so much! Your explanation is helpful. I will most likely continue posting more questions, since I'm drilling LR heavily at the moment.

  • lsat_suslsat_sus Core Member
    1417 karma

    @cat_kimball make sure you @ the person's name otherwise they won't get a notification for your response. I only saw your comment because I forgot to close this window aha.

    and PM me some questions you have cuz that'll help me and help you!! ALL THE BEST FAM!! GET DAT.

Sign In or Register to comment.